Download Homework 1, Solutions

Survey
yes no Was this document useful for you?
   Thank you for your participation!

* Your assessment is very important for improving the workof artificial intelligence, which forms the content of this project

Document related concepts
no text concepts found
Transcript
Homework 1, Solutions
(1.6) Let a, b, c ∈ Z. Use the definition of divisibility to directly prove the following.
(a) If a|b and b|c, then a|c.
(b) If a|b and b|a, then a = ±b.
(c) If a|b and a|c, then a|(b ± c).
Proof: (a)
Claim
1
2
3
4
5
Statement
a|b & b|c
∃u, v ∈ Z, b = au & c = bv
c = bv = auv
c = aw, w ∈ Z
a|c
Reason
Assumption
Definition of a|b & b|c
substituting b = au into c = bv
let w = uv.
Definition of a|c
Another way to present the reasoning: Since a|b and since b|c, by the definition of divisibility, there exist integers u and v such that b = au and c = bv. Substitute b = au into c = bv
to get c = auv. Let m = uv. Since u, v are integers, m is also an integer, and so c = am. By the
definition of divisibility, a|c.
(b) If a = 0, then a|b forces that b = 0. Hence we assume that a 6= 0 (and so b 6= 0).
Claim
1
2
3
4
5
6
7
Statement
a|b & b|a
∃u, v ∈ Z, b = au & a = bv
a = bv = auv
a(1 − uv) = 0
1 = uv
v = ±1
a = ±1
Reason
Assumption
Definition of a|b & b|a
substituting b = au into a = bv.
Algebraic manipulation.
Since a 6= 0, we can cancel a both sides of a = a(1 − uv).
u, v are integers and uv = 1.
a = bv and since v = ±1.
Another way to present the reasoning: Since a|b and b|a, by the definition of divisibility,
there exist integers u and v such that b = au and a = bv. Substitute b = au into a = bv to get
a = auv. Since a 6= 0, cancelling a both sides yields 1 = uv. Since u and v are integers, either
u = v = 1 or u = v = −1, and so v = ±1. It follows that a = bv = ±b.
(c)
1
Claim
1
2
3
4
5
Statement
a|b & a|c
∃u, v ∈ Z, b = au & c = av
b ± c = au ± av = a(u ± v)
b ± c = am
a|(b ± c)
Reason
Assumption
Definition of a|b & a|c
Algebra/Remove common factor a.
Let m = u ± v, which is also an integer.
Definition of divisibility.
Another way to present the reasoning: Since a|b and since a|c, by the definition of divisibility, there exist integers u and v such that b = au and c = av. Add (or subtract) these equalities
side by side to get b ± c = au ± av. Remove the common factor a leads to b ± c = a(u ± v).
Since u and v are integers, m = u ± v is also an integer. and so by the definition of divisibility,
a|(b ± c).
Additional Properties Proved in Class
(a) Suppose that a, b and c are integers with gcd(a, b) = 1. If a|(bc), then a|c.
(b) If p is a prime, and if a and b are integers such that p|(ab), then either p|a or p|b.
Proof: (a)
Claim
1
2
3
4
5
6
Statement
gcd(a, b) = 1
∃u, v ∈ Z, au + bv = 1
auc + bcv = c
a|auc, and a|bc
a|(auc + bcv)
a|c
Reason
Assumption
Euclidean Algorithm
multiplying c both sides of au + bv = 1
Definition of divisibility and assumption.
Exercise (1.6)(c) or Proposition 1.4(c)
Substitution of auc + bcv = c
By Euclidean Algorithm there exists integers u and v satisfying 1 = gcd(a, b) = au + bv.
Multiplying c both sides yields acu + bcv = c. Since a|auc, and a|bc, it follows by Exercise
(1.6)(c) (or Proposition 1.4(c) in the text) that a|c.
(b) We assume that p 6 |a to show that we must have p|b.
Since p is a prime and since p 6 |a, we have gcd(a, p) = 1. Therefore, by (a), gcd(p, a) = 1 and
p|(ab) implies that p|b.
(1.11) Let a and b be positive integers.
(a) Suppose that there are integers u and v satisfying au + bv = 1. Prove that gcd(a, b) = 1.
(b) Suppose that there are integers u and v satisfying au + bv = 6. Is it necessarily true that
gcd(a, b) = 6? If not, give a specific counterexample, and describe in general all of the possible
values of gcd(a, b)?
2
(c) Suppose that (u1 , v1 ) and (u2 , v2 ) are two solutions in integers to the equation au + bv = 1.
Prove that a|(v2 − v1 ) and b|(u1 − u2 ).
(d) More generally, let g = gcd(a, b) and let (u0 , v0 ) be a solution to au + bv = g. Prove that
every other solution has the form u = u0 + kb/g and v = v0 − ka/g, for some integer k.
Proofs and Solutions: (a) Let g = gcd(a, b). We want to prove that g = 1. Then by the
definition of gcd(a, b), we have g ≥ 1. It suffices to show that g ≤ 1.
By assumption, there exist integers u and v such that au + bv = 1. By the definition of
gcd, g|a and g|b, and so g|au and g|bv. By Exercise (1.6)(c) (or Proposition 1.4(c) in the text),
g|(au + bv). Since au + bv = 1, g|1, and so g ≤ 1.
Combining g ≥ 1 and g ≤ 1, we conclude that g = 1.
(b) Let a = 3 and b = 4. Then gcd(3, 4) = 1. However, for u = −6 and b = 6, we have
au + bv = 3(−6) + 4(6) = 6. In general, if gcd(a, b) = g and if for some integers u and v with
au + bv = h, then by By Exercise (1.6)(c) (or Proposition 1.4(c) in the text), we must have g|h.
Therefore, if for some integers u and v with au + bv = h, then gcd(a, b) must be a positive factor
of h.
(c)
Claim
1
2
3
4
5
6
Statement
au1 + bv1 = 1 and au2 + bv2 = 1
gcd(a, b) = 1,
a(u1 − u2 ) + b(v1 − v2 ) = 0
a(u1 − u2 ) = b(v2 − v1 )
gcd(a, b) = 1
a|(v1 − v2 ) and b|(u2 − u1 )
Reason
Assumption
Exercise (1.11)(a)
subtraction in Statement 2
Algebraic manipulation in Statement 3
Statement 2.
Additional Proposition (a)
Another way to present the reasoning: Since au1 + bv1 = 1 and au2 + bv2 = 1, subtracting
one equation from the other yields a(u1 − u2 ) + b(v1 − v2 ) = 0, or a(u1 − u2 ) = b(v2 − v1 ). Since
au1 + bv1 = 1, it follows by Exercise (1.11)(a) (just shown) that gcd(a, b) = 1. Now by Additional
Property (a) with c = (v2 − v1 ), we have a|(v2 − v1 ). Similarly (by switching a and b), b|(u1 − u2 ),
which is the same to say b|(u2 − u1 ).
(d) Let g = gcd(a, b) and let (u0 , v0 ) be a solution to au + bv = g, and let (u, v) denote the
generic solution. Since au + bv = g, it follows from (a/g)u + b/g)v = 1 and Exercise (1.11)(a)
that gcd(a/g, b/g) = 1. Therefore, both (u0 , v0 ) and (u, v) are solutions of (a/g)u + (b/g)v = 1.
By the conclusion of Exercise (1.11)(c), we have (b/g)|(u − u0 ). By the definition of divisibility,
there exists some integer k, such that u − u0 = k(b/g), or u = u0 + k(b/g).
Since (u0 , v0 ) and (u, v) are solutions to au + bv = g, we have au0 + bv0 = g, and au + bv = g.
Substitute u = u0 + k(b/g) into au + bv = g to get au0 + akb/g + bv = au0 + bv0 . Cancelling au0
both sides, we have akb/g + bv = bv0 , or bv = bv0 − bak/g,. Hence v = v0 − ka/g.
3
(1.14) Let m ≥ 1 be an integer and suppose that
a1 ≡ a2 (mod m) and b1 ≡ b2 (mod m).
Prove that
a1 ± b1 ≡ a2 ± b2 (mod m) and a1 b1 ≡ a2 b2 (mod m).
(Remark: These Properties suggest that for addition, subtraction and multiplication, doing
these operations modulo m can be done just like those for integers. )
Proof:
Claim
1
2
3
4
5
6
Claim
1
2
3
4
5
6
Statement
a1 ≡ a2 (mod m) and b1 ≡ b2 (mod m)
∃u1 ∈ Z, a1 = a2 + u1 m
∃u2 ∈ Z, b1 = b2 + u2 m
(a1 ± b1 ) = (a2 ± b2 ) + (u1 ± u2 )m
(a1 ± b1 ) = (a2 ± b2 ) + um
(a1 ± b1 ) ≡ (a2 ± b2 ) (mod m)
Statement
a1 ≡ a2 (mod m) and b1 ≡ b2 (mod m)
∃u1 ∈ Z, a1 = a2 + u1 m
∃u2 ∈ Z, b1 = b2 + u2 m
(a1 b1 ) = (a2 b2 ) + (a2 u2 + b2 u1 + u1 u2 m)m
a1 b1 = a2 b2 + um
a1 b1 ≡ a2 b2 (mod m)
Reason
Assumption
Definition of mod m congruence
Definition of mod m congruence
Combining Statements 2 and 3
Set u = u1 ± u2
Definition of mod m congruence
Reason
Assumption
Definition of mod m congruence
Definition of mod m congruence
Combining Statements 2 and 3
Set u = a2 u2 + b2 u1 + u1 u2 m
Definition of mod m congruence
(1.17) Find all values of x between 0 and m − 1 that are solutions of the following congruences.
(a) x + 17 ≡ 23 (mod 37).
(b) x + 42 ≡ 19 (mod 51).
(c) x2 ≡ 3 (mod 11).
(d) x2 ≡ 2 (mod 13).
(e) x2 ≡ 1 (mod 8).
(f) x3 − x2 + 2x − 2 ≡ 0 (mod 11).
(g) x ≡ 1 (mod 5) and x ≡ 2 (mod 7), (Find all solutions mod 35).
Solution: (a) x ≡ 23 − 17 ≡ 6 (mod 37).
(b) x ≡ 19 − 42 ≡ 19 + 51 − 42 ≡ 28 (mod 51).
(c) x2 − 3 ≡ 0 (mod 11). Since 52 ≡ 3 (mod 11), we have x2 − 3 ≡ x2 − 52 ≡ (x − 5)(x + 5) ≡ 0
(mod 11).
4
Since p = 11 is a prime, and by the property that if p is a prime, then p|ab implies p|a or p|b,
we have 11|(x − 5) or 11|(x + 5), and so x ≡ 5 or x ≡ −5 ≡ 6 (mod 11).
(d) Since
12 ≡ 122 ≡ 1, 22 ≡ 112 ≡ 4, 32 ≡ 9, 42 ≡ 92 ≡ 3, 52 ≡ 82 ≡ 12, 62 ≡ 72 ≡ 10 (mod 13),
x2 ≡ 2 (mod 13) has no solution.
(e) From (x − 1)(x + 1) ≡ 1 (mod 8), we have 8|(x − 1)(x + 1). By Exercise (1.6)(a) (with a = 2
and b = 8, c = (x − 1)(x + 1)), we have 2|(x − 1)(x + 1). Since 2 is a prime, we have x ≡ 1 (mod
2), or x ≡ −1 (mod 2). Therefore in Z8 , we have x ≡ 1, 3, 5, 7.
(f) Factoring x3 − x2 + 2x − 2 = x2 (x − 1) + 2(x − 1) = (x2 + 2)(x − 1) (mod 11). Thus 11|(x − 1),
whence x ≡ 1 (mod 11), or 11|(x2 + 2). Since x2 + 2 ≡ x2 − 9 ≡ (x − 3)(x + 2), and since 11 is
a prime, 11|(x − 3)(x + 3) implies that x ≡ 3 or x ≡ −3 ≡ 8 (mod 11). Therefore, the solutions
are x ≡ 1, 3, 8, (mod 11).
(g) Since x ≡ 1 (mod 5), we have 5|(x − 1). Among all integers between 0 and 35, we
have x ∈ {1, 6, 11, 16, 21, 26, 31}. Since x ≡ 2 (mod 7), we have 7|(x − 2). From the list
{1, 6, 11, 16, 21, 26, 31}, we single out x = 16. Therefore, the solution is x = 16.
5
Related documents